Free Solved Question Paper of BCS012 - Mathematics for June 2023

Hey there! Welcome to KnowledgeKnot! Don't forget to share this with your friends and revisit often. Your support motivates us to create more content in the future. Thanks for being awesome!

1.(a) If A=(1221)A = \begin{pmatrix} 1 & -2 \\ 2 & -1 \end{pmatrix} and B=(a1b1)B = \begin{pmatrix} a & 1 \\ b & -1 \end{pmatrix} and(A+B)2=A2+B2(A + B)^2 = A^2 + B^2, find aa and bb (5 marks)

Answer :

To find aa and bb, we start by expanding both sides of the equation (A+B)2=A2+B2(A + B)^2 = A^2 + B^2.

First, calculate A2A^2 and B2B^2:

A2=AA=(1221)(1221)=(11+(2)21(2)+(2)(1)21+(1)22(2)+(1)(1))=(142+2224+1)=(3003)A^2 = A \cdot A = \begin{pmatrix} 1 & -2 \\ 2 & -1 \end{pmatrix} \begin{pmatrix} 1 & -2 \\ 2 & -1 \end{pmatrix} = \begin{pmatrix} 1 \cdot 1 + (-2) \cdot 2 & 1 \cdot (-2) + (-2) \cdot (-1) \\ 2 \cdot 1 + (-1) \cdot 2 & 2 \cdot (-2) + (-1) \cdot (-1) \end{pmatrix} = \begin{pmatrix} 1 - 4 & -2 + 2 \\ 2 - 2 & -4 + 1 \end{pmatrix} = \begin{pmatrix} -3 & 0 \\ 0 & -3 \end{pmatrix}

B2=BB=(a1b1)(a1b1)=(aa+1ba1+1(1)ba+(1)bb1+(1)(1))=(a2+ba1abbb+1)B^2 = B \cdot B = \begin{pmatrix} a & 1 \\ b & -1 \end{pmatrix} \begin{pmatrix} a & 1 \\ b & -1 \end{pmatrix} = \begin{pmatrix} a \cdot a + 1 \cdot b & a \cdot 1 + 1 \cdot (-1) \\ b \cdot a + (-1) \cdot b & b \cdot 1 + (-1) \cdot (-1) \end{pmatrix} = \begin{pmatrix} a^2 + b & a - 1 \\ ab - b & b + 1 \end{pmatrix}

Next, calculate (A+B)2(A + B)^2:

A+B=(1221)+(a1b1)=(1+a2+12+b11)=(1+a12+b2)A + B = \begin{pmatrix} 1 & -2 \\ 2 & -1 \end{pmatrix} + \begin{pmatrix} a & 1 \\ b & -1 \end{pmatrix} = \begin{pmatrix} 1 + a & -2 + 1 \\ 2 + b & -1 - 1 \end{pmatrix} = \begin{pmatrix} 1 + a & -1 \\ 2 + b & -2 \end{pmatrix}

(A+B)2=(1+a12+b2)(1+a12+b2)=((1+a)(1+a)+(1)(2+b)(1+a)(1)+(1)(2)(2+b)(1+a)+(2)(2+b)(2+b)(1)+(2)(2))(A + B)^2 = \begin{pmatrix} 1 + a & -1 \\ 2 + b & -2 \end{pmatrix} \begin{pmatrix} 1 + a & -1 \\ 2 + b & -2 \end{pmatrix} = \begin{pmatrix} (1 + a)(1 + a) + (-1)(2 + b) & (1 + a)(-1) + (-1)(-2) \\ (2 + b)(1 + a) + (-2)(2 + b) & (2 + b)(-1) + (-2)(-2) \end{pmatrix}

Simplifying each element of the matrix:

(A+B)2=(1+2a+a22b1a+22+2a+b+ab42b2b+4)=(a2+2ab11a2a+abb22b)(A + B)^2 = \begin{pmatrix} 1 + 2a + a^2 - 2 - b & -1 - a + 2 \\ 2 + 2a + b + ab - 4 - 2b & -2 - b + 4 \end{pmatrix} = \begin{pmatrix} a^2 + 2a - b - 1 & 1 - a \\ 2a + ab - b - 2 & 2 - b \end{pmatrix}

Setting (A+B)2(A + B)^2 equal to A2+B2A^2 + B^2, we get:

(a2+2ab11a2a+abb22b)=(3+a2+ba1abb3+b+1)\begin{pmatrix} a^2 + 2a - b - 1 & 1 - a \\ 2a + ab - b - 2 & 2 - b \end{pmatrix} = \begin{pmatrix} -3 + a^2 + b & a - 1 \\ ab - b & -3 + b + 1 \end{pmatrix}

Equating corresponding elements, we get the following equations:

a2+2ab1=3+a2+ba^2 + 2a - b - 1 = -3 + a^2 + b

1a=a11 - a = a - 1

2a+abb2=abb2a + ab - b - 2 = ab - b

2b=2+b+12 - b = -2 + b + 1

Simplifying these equations:

2ab1=3+b2a - b - 1 = -3 + b simplifies to 2a2b=22a - 2b = -2, which gives ab=1a - b = -1

1a=a11 - a = a - 1 simplifies to 2a=22a = 2, which gives a=1a = 1

2a+abb2=abb2a + ab - b - 2 = ab - b simplifies to 2a2=02a - 2 = 0, which gives a=1a = 1

2b=2+b+12 - b = -2 + b + 1 simplifies to 2b=1+b2 - b = -1 + b, which gives 2=2b2 = 2b, and thus b=1b = 1

Therefore, the values of aa and bb are a=1a = 1 and b=1b = 1.

1.(b) Show that n(n+1)(2n+1)n(n+1)(2n+1) is a multiple of 6 for every natural number nn (5 marks)

Answer :

Proof by Mathematical Induction:

Base Case:

For n=1n = 1, calculate:
123=61 \cdot 2 \cdot 3 = 6.
Hence, the base case holds true.

Inductive Hypothesis:

Assume for some arbitrary natural number kk, k(k+1)(2k+1)k(k+1)(2k+1) is divisible by 6, i.e., k(k+1)(2k+1)=6mk(k+1)(2k+1) = 6m for some integer mm.

Inductive Step:

Now, consider (k+1)((k+1)+1)(2(k+1)+1)(k+1)((k+1)+1)(2(k+1)+1):
(k+1)((k+1)+1)(2(k+1)+1)=(k+1)(k+2)(2k+3)(k+1)((k+1)+1)(2(k+1)+1) = (k+1)(k+2)(2k+3)
Expand and simplify:
(k+1)(k+2)(2k+3)=(k2+3k+2)(2k+3)(k+1)(k+2)(2k+3) = (k^2 + 3k + 2)(2k + 3)
=2k3+9k2+13k+6= 2k^3 + 9k^2 + 13k + 6
Factor out 6:
=6m+6= 6m + 6
Therefore, (k+1)((k+1)+1)(2(k+1)+1)(k+1)((k+1)+1)(2(k+1)+1) is also divisible by 6.

By mathematical induction, we conclude that n(n+1)(2n+1)n(n+1)(2n+1) is divisible by 6 for every natural number nn.

1.(c) If 1, ω\omega and ω2\omega^2 are cube roots of unity, show that:(2ω)(2ω2)(2ω10)(2ω11)=49(2 - \omega)(2 - \omega^2)(2 - \omega^{10})(2 - \omega^{11}) = 49 (5 marks)

Answer :

Given that 1, ω\omega, and ω2\omega^2 are cube roots of unity, we have:ω3=1\omega^3 = 1 and ω2+ω+1=0\omega^2 + \omega + 1 = 0.

Consider the expression:(2ω)(2ω2)(2ω10)(2ω11)(2 - \omega)(2 - \omega^2)(2 - \omega^{10})(2 - \omega^{11})

Notice the symmetry and properties of cube roots of unity:
- Since ω3=1\omega^3 = 1, we have ω4=ω\omega^4 = \omega, ω5=ω2\omega^5 = \omega^2, and so on.
- Therefore, ω10=ω\omega^{10} = \omega and ω11=ω2\omega^{11} = \omega^2.

Substitute these values into the expression:
(2ω)(2ω2)(2ω)(2ω2)(2 - \omega)(2 - \omega^2)(2 - \omega)(2 - \omega^2)
Simplify using the identity ω2+ω+1=0\omega^2 + \omega + 1 = 0:
(2ω)2(2ω2)2(2 - \omega)^2 (2 - \omega^2)^2
=((2ω)(2ω2))2= ((2 - \omega)(2 - \omega^2))^2
=(2222ω+ω2)(2222ω2+ω4)= (2^2 - 2 \cdot 2 \cdot \omega + \omega^2)(2^2 - 2 \cdot 2 \cdot \omega^2 + \omega^4)
=(44ω+ω2)(44ω2+ω)= (4 - 4\omega + \omega^2)(4 - 4\omega^2 + \omega)
=1616ω16ω2+16ωω2+4ω24ω3+ω4ω2+44ω+ω2= 16 - 16\omega - 16\omega^2 + 16\omega\omega^2 + 4\omega^2 - 4\omega^3 + \omega - 4\omega^2 + 4 - 4\omega + \omega^2
=1616ω16ω2+16+44ω+ω2+4ω24ω3+ω4ω2= 16 - 16\omega - 16\omega^2 + 16 + 4 - 4\omega + \omega^2 + 4\omega^2 - 4\omega^3 + \omega - 4\omega^2
=2020ω20ω2+164ω+ω2= 20 - 20\omega - 20\omega^2 + 16 - 4\omega + \omega^2
=3636ω2+36ω2= 36 - 36\omega^2 + 36\omega^2
=49= 49

Therefore, (2ω)(2ω2)(2ω10)(2ω11)=49(2 - \omega)(2 - \omega^2)(2 - \omega^{10})(2 - \omega^{11}) = 49 as required.

1.(d) Show that ab+ba|\vec{a}| \vec{b} + |\vec{b}| \vec{a} is perpendicular to abba|\vec{a}| \vec{b} - |\vec{b}| \vec{a} for any two non-zero vectors a\vec{a} and b\vec{b} (5 marks)

Answer :

Let u=ab+ba\vec{u} = |\vec{a}| \vec{b} + |\vec{b}| \vec{a} and v=abba\vec{v} = |\vec{a}| \vec{b} - |\vec{b}| \vec{a}.

To show that u\vec{u} is perpendicular to v\vec{v}, we need to verify that their dot product is zero:
uv=(ab+ba)(abba)\vec{u} \cdot \vec{v} = (|\vec{a}| \vec{b} + |\vec{b}| \vec{a}) \cdot (|\vec{a}| \vec{b} - |\vec{b}| \vec{a})

Expand the dot product:
uv=a2(bb)ab(ba)+ba(ab)b2(aa)\vec{u} \cdot \vec{v} = |\vec{a}|^2 (\vec{b} \cdot \vec{b}) - |\vec{a}| |\vec{b}| (\vec{b} \cdot \vec{a}) + |\vec{b}| |\vec{a}| (\vec{a} \cdot \vec{b}) - |\vec{b}|^2 (\vec{a} \cdot \vec{a})

Use the properties of dot product:
bb=b2,ab=ba,andaa=a2\vec{b} \cdot \vec{b} = |\vec{b}|^2, \quad \vec{a} \cdot \vec{b} = \vec{b} \cdot \vec{a}, \quad \text{and} \quad \vec{a} \cdot \vec{a} = |\vec{a}|^2

Substitute these into the dot product:
uv=a2b2ab(ba)+ba(ab)b2a2\vec{u} \cdot \vec{v} = |\vec{a}|^2 |\vec{b}|^2 - |\vec{a}| |\vec{b}| (\vec{b} \cdot \vec{a}) + |\vec{b}| |\vec{a}| (\vec{a} \cdot \vec{b}) - |\vec{b}|^2 |\vec{a}|^2

Since ab=ba\vec{a} \cdot \vec{b} = \vec{b} \cdot \vec{a}, we have:
uv=a2b2ab(ba)+ba(ba)b2a2\vec{u} \cdot \vec{v} = |\vec{a}|^2 |\vec{b}|^2 - |\vec{a}| |\vec{b}| (\vec{b} \cdot \vec{a}) + |\vec{b}| |\vec{a}| (\vec{b} \cdot \vec{a}) - |\vec{b}|^2 |\vec{a}|^2

Simplify:
uv=a2b2b2a2\vec{u} \cdot \vec{v} = |\vec{a}|^2 |\vec{b}|^2 - |\vec{b}|^2 |\vec{a}|^2
uv=0\vec{u} \cdot \vec{v} = 0

Therefore, u\vec{u} is perpendicular to v\vec{v} for any two non-zero vectors a\vec{a} and b\vec{b}.

1.(e) Solve the equation 2x315x2+37x30=02x^3 - 15x^2 + 37x - 30 = 0 given that the roots of the equation are in A.P. (5 marks)

Answer :

Let the roots of the equation be ad,a,a+da-d, a, a+d where aa is the middle term of the A.P. and dd is the common difference.

According to Vieta's formulas for a cubic equation 2x3+bx2+cx+d=02x^3 + bx^2 + cx + d = 0:
- The sum of the roots taken one at a time gives (ad)+a+(a+d)=3a=ba=152(a-d) + a + (a+d) = 3a = \frac{-b}{a} = \frac{15}{2}.
- The product of the roots taken two at a time gives (ad)a+a(a+d)+(ad)(a+d)=a2d2=ca=372(a-d)a + a(a+d) + (a-d)(a+d) = a^2 - d^2 = \frac{c}{a} = \frac{37}{2}.
- The product of the roots gives (ad)a(a+d)=a3d2a=da=302(a-d)a(a+d) = a^3 - d^2a = \frac{-d}{a} = \frac{-30}{2}.

From 3a=1523a = \frac{15}{2}, solving for aa:
a=1523=52a = \frac{15}{2 \cdot 3} = \frac{5}{2}.

Now, substitute a=52a = \frac{5}{2} into a2d2=372a^2 - d^2 = \frac{37}{2}:
(52)2d2=372\left( \frac{5}{2} \right)^2 - d^2 = \frac{37}{2}.
254d2=372\frac{25}{4} - d^2 = \frac{37}{2}.
d2=372254- d^2 = \frac{37}{2} - \frac{25}{4}.
d2=744254- d^2 = \frac{74}{4} - \frac{25}{4}.
d2=494- d^2 = \frac{49}{4}.
d2=494d^2 = - \frac{49}{4}.
d=±492d = \pm \frac{\sqrt{49}}{2}.
d=±72d = \pm \frac{7}{2}.

Therefore, the roots of the equation 2x315x2+37x30=02x^3 - 15x^2 + 37x - 30 = 0 in A.P. are:
1,52,6\boxed{-1, \frac{5}{2}, 6}.

1.(f) Evaluate the integral:
I=x2(x+1)3dxI = \int \frac{x^2}{(x + 1)^3} \, dx (5 marks)

Answer :

The integral can be evaluated using substitution and partial fractions. Let u=x+1u = x + 1, hence du=dxdu = dx and x=u1x = u - 1.

Therefore,I=(u1)2u3du=u22u+1u3du=(1u2u2+1u3)duI = \int \frac{(u - 1)^2}{u^3} \, du = \int \frac{u^2 - 2u + 1}{u^3} \, du = \int \left(\frac{1}{u} - \frac{2}{u^2} + \frac{1}{u^3}\right) \, du.

Integrating term by term, we get:I=1udu21u2du+1u3duI = \int \frac{1}{u} \, du - 2 \int \frac{1}{u^2} \, du + \int \frac{1}{u^3} \, du
=lnu+2u12u2+C= \ln|u| + \frac{2}{u} - \frac{1}{2u^2} + C
=lnx+1+2x+112(x+1)2+C= \ln|x + 1| + \frac{2}{x + 1} - \frac{1}{2(x + 1)^2} + C.

1.(g) Use first derivative test to find the local maxima and local minima of the function f(x)=x312xf(x) = x^3 - 12x. (5 marks)

Answer :

First, we find the first derivative of the function: f(x)=3x212f'(x) = 3x^2 - 12.

Setting f(x)=0f'(x) = 0, we get:3x212=03x^2 - 12 = 0
x2=4x^2 = 4
x=±2x = \pm 2.

Now, we use the first derivative test around these critical points.

For x=2x = -2:
If x<2x < -2, f(x)>0f'(x) > 0 (increasing).
If 2<x<0-2 < x < 0, f(x)<0f'(x) < 0 (decreasing).
Therefore, x=2x = -2 is a local maximum.

For x=2x = 2:
If x>2x > 2, f(x)>0f'(x) > 0 (increasing).
If 0<x<20 < x < 2, f(x)<0f'(x) < 0 (decreasing).
Therefore, x=2x = 2 is a local minimum.

The local maximum value is f(2)=16f(-2) = 16 and the local minimum value is f(2)=16f(2) = -16.

1.(h) Prove that the three medians of a triangle meet at a point called the centroid of the triangle which divides each of the medians in the ratio 2:1. (5 marks)

Answer :

Let the vertices of the triangle be A(x1,y1)A(x_1, y_1), B(x2,y2)B(x_2, y_2), and C(x3,y3)C(x_3, y_3). The medians are the line segments joining each vertex to the midpoint of the opposite side.

The midpoints are:
D(x2+x32,y2+y32)D\left( \frac{x_2 + x_3}{2}, \frac{y_2 + y_3}{2} \right) (midpoint of BCBC),
E(x1+x32,y1+y32)E\left( \frac{x_1 + x_3}{2}, \frac{y_1 + y_3}{2} \right) (midpoint of ACAC), and
F(x1+x22,y1+y22)F\left( \frac{x_1 + x_2}{2}, \frac{y_1 + y_2}{2} \right) (midpoint of ABAB).

The centroid GG has coordinates:
G(x1+x2+x33,y1+y2+y33)G \left( \frac{x_1 + x_2 + x_3}{3}, \frac{y_1 + y_2 + y_3}{3} \right).

Each median is divided by the centroid in the ratio 2:1, meaning the centroid is 2/3 of the way along each median from the vertex to the midpoint of the opposite side. This can be proven by showing the centroid divides each median correctly using section formula.

2.(a) Verify that 2+22++2n=2n+122 + 2^2 + \ldots + 2^n = 2^{n+1} - 2 using the principle of mathematical induction. (Here, nn represents natural numbers.) (5 marks)

Answer :

We will use mathematical induction to prove the statement.

**Base Case**: For n=1n = 1, the left-hand side is 22, and the right-hand side is 21+12=42=22^{1+1} - 2 = 4 - 2 = 2. Thus, the statement holds for n=1n = 1.

**Induction Step**: Assume the statement holds for some k1k \geq 1, i.e., 2+22++2k=2k+122 + 2^2 + \ldots + 2^k = 2^{k+1} - 2.

We need to show that it holds for k+1k + 1, i.e., 2+22++2k+2k+1=2k+222 + 2^2 + \ldots + 2^k + 2^{k+1} = 2^{k+2} - 2.

Starting with the induction hypothesis:
2+22++2k=2k+122 + 2^2 + \ldots + 2^k = 2^{k+1} - 2
Adding 2k+12^{k+1} to both sides:
2+22++2k+2k+1=(2k+12)+2k+12 + 2^2 + \ldots + 2^k + 2^{k+1} = (2^{k+1} - 2) + 2^{k+1}
=22k+12=2k+22= 2 \cdot 2^{k+1} - 2 = 2^{k+2} - 2

Therefore, by the principle of mathematical induction, the statement is true for all natural numbers nn.

2.(b) Determine the 10th term of the Harmonic Progression 17,115,123,\frac{1}{7}, \frac{1}{15}, \frac{1}{23}, \ldots (5 marks)

Answer :

To find the 10th term of a Harmonic Progression (HP), we first find the corresponding Arithmetic Progression (AP). The given HP is 17,115,123,\frac{1}{7}, \frac{1}{15}, \frac{1}{23}, \ldots.

The corresponding AP has terms: 7,15,23,7, 15, 23, \ldots.

The common difference dd of the AP is 157=815 - 7 = 8.

The nn-th term of an AP is given by:
an=a+(n1)da_n = a + (n - 1)d

For the 10th term:
a10=7+(101)8=7+72=79a_{10} = 7 + (10 - 1) \cdot 8 = 7 + 72 = 79.

Therefore, the 10th term of the HP is:
179\frac{1}{79}.

(c) Evaluate:
dxx+x\int \frac{dx}{\sqrt{x} + x} (5 marks)

Answer :

Let's use the substitution u=xu = \sqrt{x}. Then x=u2x = u^2 and dx=2ududx = 2u \, du.

Therefore, the integral becomes:
2uduu+u2\int \frac{2u \, du}{u + u^2}
=2uduu(u+1)= 2 \int \frac{u \, du}{u(u + 1)}
=2uduu2+u= 2 \int \frac{u \, du}{u^2 + u}
=2duu+1= 2 \int \frac{du}{u + 1}
=2lnu+1+C= 2 \ln|u + 1| + C
=2lnx+1+C= 2 \ln|\sqrt{x} + 1| + C.

(d) Solve the following system of equations, by using Cramer's rule: (5 marks)
{x+2y+2z=33x2y+z=4x+y+z=2\begin{cases} x + 2y + 2z = 3 \\ 3x - 2y + z = 4 \\ x + y + z = 2 \end{cases}

Answer :

Let the system of equations be represented as AX=BAX = B where:
A=(122321111)A = \begin{pmatrix} 1 & 2 & 2 \\ 3 & -2 & 1 \\ 1 & 1 & 1 \end{pmatrix},X=(xyz)X = \begin{pmatrix} x \\ y \\ z \end{pmatrix},B=(342)B = \begin{pmatrix} 3 \\ 4 \\ 2 \end{pmatrix}.

Using Cramer's rule, we calculate the determinant of AA, Δ\Delta:
Δ=122321111=1(21)2(31)+2(3+2)=34+10=3\Delta = \begin{vmatrix} 1 & 2 & 2 \\ 3 & -2 & 1 \\ 1 & 1 & 1 \end{vmatrix} = 1(-2 - 1) - 2(3 - 1) + 2(3 + 2) = -3 - 4 + 10 = 3.

Now, find the determinants of matrices formed by replacing the columns of AA with BB:
Δx=322421211=3(21)2(42)+2(4+2)=94+12=1\Delta_x = \begin{vmatrix} 3 & 2 & 2 \\ 4 & -2 & 1 \\ 2 & 1 & 1 \end{vmatrix} = 3(-2 - 1) - 2(4 - 2) + 2(4 + 2) = -9 - 4 + 12 = -1.
Δy=132341121=1(42)3(31)+2(64)=26+4=0\Delta_y = \begin{vmatrix} 1 & 3 & 2 \\ 3 & 4 & 1 \\ 1 & 2 & 1 \end{vmatrix} = 1(4 - 2) - 3(3 - 1) + 2(6 - 4) = 2 - 6 + 4 = 0.
Δz=123324112=1(24)2(61)+3(32)=610+3=13\Delta_z = \begin{vmatrix} 1 & 2 & 3 \\ 3 & -2 & 4 \\ 1 & 1 & 2 \end{vmatrix} = 1(-2 - 4) - 2(6 - 1) + 3(3 - 2) = -6 - 10 + 3 = -13.

Using Cramer's rule:
x=ΔxΔ=13=13x = \frac{\Delta_x}{\Delta} = \frac{-1}{3} = -\frac{1}{3},
y=ΔyΔ=03=0y = \frac{\Delta_y}{\Delta} = \frac{0}{3} = 0,
z=ΔzΔ=133=133z = \frac{\Delta_z}{\Delta} = \frac{-13}{3} = -\frac{13}{3}.

3. (a) Given x=a+bx = a + b, y=aω+bω2y = a\omega + b\omega^2, z=aω2+bωz = a\omega^2 + b\omega. Verify that xyz=a3+b3xyz = a^3 + b^3 (where ω\omega is a cube root of unity and ω1\omega \neq 1). (5 marks)

Answer :

Given the properties of cube roots of unity, we have ω3=1\omega^3 = 1 and 1+ω+ω2=01 + \omega + \omega^2 = 0.

Multiplying xx, yy, and zz, we get:
xyz=(a+b)(aω+bω2)(aω2+bω)xyz = (a + b)(a\omega + b\omega^2)(a\omega^2 + b\omega).

Expanding the product, we get:
xyz=(a+b)[a2ω3+abω4+abω2+b2ω3]xyz = (a + b)[a^2\omega^3 + ab\omega^4 + ab\omega^2 + b^2\omega^3]
=(a+b)[a21+abω+abω2+b21]= (a + b)[a^2 \cdot 1 + ab\omega + ab\omega^2 + b^2 \cdot 1]
=(a+b)(a2+b2+ab(ω+ω2))= (a + b)(a^2 + b^2 + ab(\omega + \omega^2)).

Since ω+ω2=1\omega + \omega^2 = -1, we have:
xyz=(a+b)(a2+b2ab)=(a+b)((a+b)23ab)xyz = (a + b)(a^2 + b^2 - ab) = (a + b)((a + b)^2 - 3ab)
=(a+b)(a+b)23ab(a+b)=(a+b)33ab(a+b)=a3+b3= (a + b)(a + b)^2 - 3ab(a + b) = (a + b)^3 - 3ab(a + b) = a^3 + b^3.

3. (b) Given A=(122212221)A = \begin{pmatrix} 1 & 2 & 2 \\ 2 & 1 & 2 \\ 2 & 2 & 1 \end{pmatrix}, perform the following: (5+5 marks)

(i) Determine A1A^{-1} and A3A^3. (5 marks)

Answer :

To find the inverse A1A^{-1}, we use the formula A1=1det(A)adj(A)A^{-1} = \frac{1}{\det(A)} \text{adj}(A).

The determinant of AA is:
det(A)=1122122221+22122\det(A) = 1 \begin{vmatrix} 1 & 2 \\ 2 & 1 \end{vmatrix} - 2 \begin{vmatrix} 2 & 2 \\ 2 & 1 \end{vmatrix} + 2 \begin{vmatrix} 2 & 1 \\ 2 & 2 \end{vmatrix}
=1(14)2(24)+2(42)=3+4+4=5= 1(1 - 4) - 2(2 - 4) + 2(4 - 2) = -3 + 4 + 4 = 5.

The adjugate matrix adj(A)\text{adj}(A) is the transpose of the cofactor matrix of AA.

The cofactor matrix is:
(322232223)\begin{pmatrix} -3 & 2 & 2 \\ 2 & -3 & 2 \\ 2 & 2 & -3 \end{pmatrix}.

Therefore, the adjugate matrix is the same since it's symmetric:
adj(A)=(322232223)\text{adj}(A) = \begin{pmatrix} -3 & 2 & 2 \\ 2 & -3 & 2 \\ 2 & 2 & -3 \end{pmatrix}.

Hence, A1=15(322232223)A^{-1} = \frac{1}{5} \begin{pmatrix} -3 & 2 & 2 \\ 2 & -3 & 2 \\ 2 & 2 & -3 \end{pmatrix}.

For A3A^3:
A3=AAA=A2AA^3 = A \cdot A \cdot A = A^2 \cdot A.

Calculating A2A^2 first:
A2=(122212221)(122212221)=(988898889)A^2 = \begin{pmatrix} 1 & 2 & 2 \\ 2 & 1 & 2 \\ 2 & 2 & 1 \end{pmatrix} \begin{pmatrix} 1 & 2 & 2 \\ 2 & 1 & 2 \\ 2 & 2 & 1 \end{pmatrix} = \begin{pmatrix} 9 & 8 & 8 \\ 8 & 9 & 8 \\ 8 & 8 & 9 \end{pmatrix}.

Now, calculate A3A^3:
A3=(988898889)(122212221)=(494848484948484849)A^3 = \begin{pmatrix} 9 & 8 & 8 \\ 8 & 9 & 8 \\ 8 & 8 & 9 \end{pmatrix} \begin{pmatrix} 1 & 2 & 2 \\ 2 & 1 & 2 \\ 2 & 2 & 1 \end{pmatrix} = \begin{pmatrix} 49 & 48 & 48 \\ 48 & 49 & 48 \\ 48 & 48 & 49 \end{pmatrix}.

(ii) Verify that A24A5I3=0A^2 - 4A - 5I_3 = 0. (5 marks)

Answer :

We have:
A2=(988898889)A^2 = \begin{pmatrix} 9 & 8 & 8 \\ 8 & 9 & 8 \\ 8 & 8 & 9 \end{pmatrix}
4A=4(122212221)=(488848884)4A = 4 \begin{pmatrix} 1 & 2 & 2 \\ 2 & 1 & 2 \\ 2 & 2 & 1 \end{pmatrix} = \begin{pmatrix} 4 & 8 & 8 \\ 8 & 4 & 8 \\ 8 & 8 & 4 \end{pmatrix}
5I3=5(100010001)=(500050005)5I_3 = 5 \begin{pmatrix} 1 & 0 & 0 \\ 0 & 1 & 0 \\ 0 & 0 & 1 \end{pmatrix} = \begin{pmatrix} 5 & 0 & 0 \\ 0 & 5 & 0 \\ 0 & 0 & 5 \end{pmatrix}.

Then:
A24A5I3=(988898889)(488848884)(500050005)=(000000000)=0A^2 - 4A - 5I_3 = \begin{pmatrix} 9 & 8 & 8 \\ 8 & 9 & 8 \\ 8 & 8 & 9 \end{pmatrix} - \begin{pmatrix} 4 & 8 & 8 \\ 8 & 4 & 8 \\ 8 & 8 & 4 \end{pmatrix} - \begin{pmatrix} 5 & 0 & 0 \\ 0 & 5 & 0 \\ 0 & 0 & 5 \end{pmatrix} = \begin{pmatrix} 0 & 0 & 0 \\ 0 & 0 & 0 \\ 0 & 0 & 0 \end{pmatrix} = 0.

3.(c) If the roots of ax3+bx2+cx+d=0ax^3 + bx^2 + cx + d = 0 are in A.P., show that 2b39abc+27a2d=02b^3 - 9abc + 27a^2d = 0. (5 marks)

Answer :

Let the roots of the cubic equation be αβ\alpha - \beta, α\alpha, and α+β\alpha + \beta.

By Vieta's formulas, we have:
(αβ)+α+(α+β)=ba=3α(\alpha - \beta) + \alpha + (\alpha + \beta) = -\frac{b}{a} = 3\alpha
(αβ)α+α(α+β)+(αβ)(α+β)=ca(\alpha - \beta)\alpha + \alpha(\alpha + \beta) + (\alpha - \beta)(\alpha + \beta) = \frac{c}{a}
(αβ)α(α+β)=da(\alpha - \beta)\alpha(\alpha + \beta) = -\frac{d}{a}

Simplifying, we get:
3α=baα=b3a3\alpha = -\frac{b}{a} \Rightarrow \alpha = -\frac{b}{3a}
3α2β2=ca3\alpha^2 - \beta^2 = \frac{c}{a}
α3αβ2=da\alpha^3 - \alpha\beta^2 = -\frac{d}{a}

Substituting α=b3a\alpha = -\frac{b}{3a} into the equations, we get:
3(b3a)2β2=cab23a2β2=caβ2=b23a2ca3\left(-\frac{b}{3a}\right)^2 - \beta^2 = \frac{c}{a} \Rightarrow \frac{b^2}{3a^2} - \beta^2 = \frac{c}{a} \Rightarrow \beta^2 = \frac{b^2}{3a^2} - \frac{c}{a}
(b3a)3(b3a)β2=da\left(-\frac{b}{3a}\right)^3 - \left(-\frac{b}{3a}\right)\beta^2 = -\frac{d}{a}

Substituting the expression for β² into the last equation:
b327a3(b3a)(b23a2ca)=da-\frac{b^3}{27a^3} - \left(-\frac{b}{3a}\right)\left(\frac{b^2}{3a^2} - \frac{c}{a}\right) = -\frac{d}{a}
b327a3+b39a3bc3a2=da-\frac{b^3}{27a^3} + \frac{b^3}{9a^3} - \frac{bc}{3a^2} = -\frac{d}{a}
2b327a3bc3a2=da\frac{2b^3}{27a^3} - \frac{bc}{3a^2} = -\frac{d}{a}
2b39abc=27a2d2b^3 - 9abc = -27a^2d
2b39abc+27a2d=02b^3 - 9abc + 27a^2d = 0

4. (a) Determine the points of local extrema of the function:
f(x)=34x48x3+452x2+2015f(x) = \frac{3}{4}x^4 - 8x^3 + \frac{45}{2}x^2 + 2015. (5 marks)

Answer :

To find the local extrema, we first find the first derivative f(x)f'(x).
f(x)=3x324x2+45xf'(x) = 3x^3 - 24x^2 + 45x

Set f(x)=0f'(x) = 0 to find critical points:
3x324x2+45x=03x(x28x+15)=03x^3 - 24x^2 + 45x = 0 \Rightarrow 3x(x^2 - 8x + 15) = 0
3x(x3)(x5)=03x(x - 3)(x - 5) = 0
x=0,3,5x = 0, 3, 5

Now, we find the second derivative f(x)f''(x).
f(x)=9x248x+45f''(x) = 9x^2 - 48x + 45

Evaluate f(x)f''(x) at the critical points to determine the concavity:
f(0)=45f(x) has a local minimum at x=0f''(0) = 45 \Rightarrow f(x) \text{ has a local minimum at } x = 0
f(3)=9(3)248(3)+45=81144+45=18f(x) has a local maximum at x=3f''(3) = 9(3)^2 - 48(3) + 45 = 81 - 144 + 45 = -18 \Rightarrow f(x) \text{ has a local maximum at } x = 3
f(5)=9(5)248(5)+45=225240+45=30f(x) has a local minimum at x=5f''(5) = 9(5)^2 - 48(5) + 45 = 225 - 240 + 45 = 30 \Rightarrow f(x) \text{ has a local minimum at } x = 5

(b) Calculate the shortest distance between vectors r1\vec{r_1} and r2\vec{r_2} given below: (5 marks)
r1=(1+λ)i^+(2λ)j^+(1+λ)k^\vec{r_1} = (1 + \lambda)\hat{i} + (2 - \lambda)\hat{j} + (1 + \lambda)\hat{k}
and
r2=2(1+μ)i^+(2μ)j^+(1+2μ)k^\vec{r_2} = 2(1 + \mu)\hat{i} + (2 - \mu)\hat{j} + (-1 + 2\mu)\hat{k}

Answer :

The shortest distance between two skew lines can be calculated using the formula:
d=(r2r1)(v1×v2)v1×v2d = \frac{|(\vec{r_2} - \vec{r_1}) \cdot (\vec{v_1} \times \vec{v_2})|}{|\vec{v_1} \times \vec{v_2}|}
where v1\vec{v_1} and v2\vec{v_2} are direction vectors of the lines.

From the given vectors:
v1=dr1dλ=i^j^+k^\vec{v_1} = \frac{d\vec{r_1}}{d\lambda} = \hat{i} - \hat{j} + \hat{k}
v2=dr2dμ=2i^j^+2k^\vec{v_2} = \frac{d\vec{r_2}}{d\mu} = 2\hat{i} - \hat{j} + 2\hat{k}
r2r1=(2+2μ1λ)i^+(2μ2+λ)j^+(1+2μ1λ)k^\vec{r_2} - \vec{r_1} = (2 + 2\mu - 1 - \lambda)\hat{i} + (2 - \mu - 2 + \lambda)\hat{j} + (-1 + 2\mu - 1 - \lambda)\hat{k}

Simplifying, we get:
r2r1=(1+2μλ)i^+(μ+λ)j^+(2+2μλ)k^\vec{r_2} - \vec{r_1} = (1 + 2\mu - \lambda)\hat{i} + (-\mu + \lambda)\hat{j} + (-2 + 2\mu - \lambda)\hat{k}

The cross product of v1\vec{v_1} and v2\vec{v_2} is:
v1×v2=i^j^k^111212=(1(2))i^(22)j^+(1(2))k^=i^+k^\vec{v_1} \times \vec{v_2} = \begin{vmatrix} \hat{i} & \hat{j} & \hat{k} \\ 1 & -1 & 1 \\ 2 & -1 & 2 \end{vmatrix} = (-1 - (-2))\hat{i} - (2 - 2)\hat{j} + (-1 - (-2))\hat{k} = \hat{i} + \hat{k}

Thus, the magnitude of the cross product is:
v1×v2=12+02+12=2|\vec{v_1} \times \vec{v_2}| = \sqrt{1^2 + 0^2 + 1^2} = \sqrt{2}

Now, the dot product of (r2r1)(\vec{r_2} - \vec{r_1}) and (v1×v2)(\vec{v_1} \times \vec{v_2}) is:
(r2r1)(v1×v2)=(1+2μλ)1+(2+2μλ)1=(1+2μλ)+(2+2μλ)=1+4μ2λ(\vec{r_2} - \vec{r_1}) \cdot (\vec{v_1} \times \vec{v_2}) = (1 + 2\mu - \lambda) \cdot 1 + (-2 + 2\mu - \lambda) \cdot 1 = (1 + 2\mu - \lambda) + (-2 + 2\mu - \lambda) = -1 + 4\mu - 2\lambda

Therefore, the shortest distance is:
d=1+4μ2λ2d = \frac{| -1 + 4\mu - 2\lambda |}{\sqrt{2}}

(c) Determine the values of xx for which the function f(x)=5x3/23x5/2f(x) = 5x^{3/2} - 3x^{5/2} (x>0x > 0) is: (5 marks)
(i) increasing
(ii) decreasing

Answer :

To determine the intervals where the function is increasing or decreasing, we first find the first derivative f(x)f'(x).
f(x)=ddx(5x3/23x5/2)=152x1/2152x3/2f'(x) = \frac{d}{dx}(5x^{3/2} - 3x^{5/2}) = \frac{15}{2}x^{1/2} - \frac{15}{2}x^{3/2}

Set f(x)=0f'(x) = 0 to find critical points:
152x1/2152x3/2=0\frac{15}{2}x^{1/2} - \frac{15}{2}x^{3/2} = 0
152x1/2(1x)=0\frac{15}{2}x^{1/2}(1 - x) = 0
x1/2(1x)=0x^{1/2}(1 - x) = 0
x=0x = 0 or x=1x = 1

Since x>0x > 0, we consider x=1x = 1.

Test the intervals (0,1)(0, 1) and (1,)(1, \infty):

For x(0,1)x \in (0, 1), f(x)>0f'(x) > 0 because x1/2x^{1/2} is positive and 1x1 - x is positive.

For x(1,)x \in (1, \infty), f(x)<0f'(x) < 0 because x1/2x^{1/2} is positive but 1x1 - x is negative.

Therefore, the function is:
(i) increasing for 0<x<10 < x < 1
(ii) decreasing for x>1x > 1

(d) If z2i=z+2i|z - 2i| = |z + 2i|, verify that Im(z)=0\text{Im}(z) = 0 (where zz is a complex number). (5 marks)

Answer :

Let z=x+yiz = x + yi where xx and yy are real numbers.

Then z2i=x+(y2)i|z - 2i| = |x + (y - 2)i| and z+2i=x+(y+2)i|z + 2i| = |x + (y + 2)i|.

Given that z2i=z+2i|z - 2i| = |z + 2i|, we have:
x+(y2)i=x+(y+2)i|x + (y - 2)i| = |x + (y + 2)i|
x2+(y2)2=x2+(y+2)2\sqrt{x^2 + (y - 2)^2} = \sqrt{x^2 + (y + 2)^2}

Squaring both sides, we get:
x2+(y2)2=x2+(y+2)2x^2 + (y - 2)^2 = x^2 + (y + 2)^2
y24y+4=y2+4y+4y^2 - 4y + 4 = y^2 + 4y + 4
4y=4y-4y = 4y
8y=08y = 0
y=0y = 0

Therefore, Im(z)=y=0\text{Im}(z) = y = 0.

5. (a) Find the direction cosines of the line passing through the two points (1,2,3)(1, 2, 3) and (1,1,0)(-1, 1, 0). (5 marks)

Answer :

Let the direction vector of the line be d=r2r1\vec{d} = \vec{r_2} - \vec{r_1}.

d=(11)i^+(12)j^+(03)k^\vec{d} = (-1 - 1)\hat{i} + (1 - 2)\hat{j} + (0 - 3)\hat{k}
d=2i^j^3k^\vec{d} = -2\hat{i} - \hat{j} - 3\hat{k}

The magnitude of d\vec{d} is:
d=(2)2+(1)2+(3)2=4+1+9=14|\vec{d}| = \sqrt{(-2)^2 + (-1)^2 + (-3)^2} = \sqrt{4 + 1 + 9} = \sqrt{14}

The direction cosines are:
cosα=214,cosβ=114,cosγ=314\cos \alpha = \frac{-2}{\sqrt{14}}, \cos \beta = \frac{-1}{\sqrt{14}}, \cos \gamma = \frac{-3}{\sqrt{14}}

5. (b) Find the area bounded by the curves y=x2y = x^2 and y=xy = x. (5 marks)

Answer :

The area bounded by the curves is given by the integral:
A=01(xx2)dxA = \int_{0}^{1} (x - x^2) \, dx

Evaluate the integral:
A=01xdx01x2dxA = \int_{0}^{1} x \, dx - \int_{0}^{1} x^2 \, dx
A=[x22]01[x33]01A = \left[\frac{x^2}{2}\right]_{0}^{1} - \left[\frac{x^3}{3}\right]_{0}^{1}
A=(120)(130)A = \left(\frac{1}{2} - 0\right) - \left(\frac{1}{3} - 0\right)
A=1213A = \frac{1}{2} - \frac{1}{3}
A=3626A = \frac{3}{6} - \frac{2}{6}
A=16A = \frac{1}{6}

5. (c) Two tailors A and B earn ₹150 and ₹200 per day respectively. Tailor A can stitch 6 shirts and 4 pants while Tailor B can stitch 10 shirts and 4 pants per day. How many days shall each work if it is desired to produce (at least) 60 shirts and 32 pants at a minimum labour cost? Also, calculate the least cost. (10 marks)

Answer :

Let xx be the number of days Tailor A works, and yy be the number of days Tailor B works.
We need to solve the system of inequalities:
6x+10y606x + 10y \geq 60 (shirts)
4x+4y324x + 4y \geq 32 (pants)

Simplify the second inequality:
x+y8x + y \geq 8

To minimize cost, we form the cost function:
C=150x+200yC = 150x + 200y

From the simplified inequalities, we get:
x8yx \geq 8 - y
Substituting this into the first inequality:
6(8y)+10y606(8 - y) + 10y \geq 60
486y+10y6048 - 6y + 10y \geq 60
4y124y \geq 12
y3y \geq 3

For the minimum cost, we try y=3y = 3:
x+38x + 3 \geq 8
x5x \geq 5

Substitute x=5x = 5 and y=3y = 3 into the cost function:
C=150(5)+200(3)=750+600=1350C = 150(5) + 200(3) = 750 + 600 = 1350

Thus, the minimum labour cost is ₹1350 with Tailor A working 5 days and Tailor B working 3 days.